Можно ли вообще при заданных матрицах плотности подсистем восстановить матрицу плотности всей системы?

Рассмотрим двудольную систему в запутанном смешанном состоянии. Поскольку его матрицу плотности всегда можно диагонализовать, мы можем записать ее следующим образом:

(1) р А Б "=" Дж п Дж | ψ Дж ψ Дж | "=" я , к С я , к | а я а я | | б к б к | ,

где { | ψ Дж } является ортонормированным базисом в ЧАС А Б "=" ЧАС А ЧАС Б , и ортоноральные базисы в ЧАС А и ЧАС Б даны { | а я } и { | б Дж } .

Можно получить приведенные матрицы плотности для подсистем A и B:

(2) р А "=" Т р Б ( р А Б ) "=" я , к С я , к | а я а я | р Б "=" я , к С я , к | б к б к |

Итак, мой вопрос: учитывая спектры (наборы собственных значений) р А и р Б , можно ли вообще восстановить спектр р А Б ? Если нет, то стоит ли за этим результатом какая-то физическая интуиция? Можно ли обобщить этот результат на любую многочастную систему?

В качестве игрушечной модели я рассматривал г я м ( ЧАС А ) "=" 2 ; и г я м ( ЧАС Б ) "=" 3 . После применения (1) и (2) и с учетом того, что следы всех матриц плотности "=" 1 , я пришел к системе уравнений с числом неизвестных больше числа независимых уравнений. Поэтому, если я не упускаю некоторые другие ограничения, эта задача, похоже, не имеет решения в целом.

РЕДАКТИРОВАТЬ:

Я хочу исправить моменты, упомянутые Эмилио Писанти и Люзанн.
(По крайней мере) одна вещь, которую я упустил из виду, это то, что для произвольного ортонормированного базиса { | ψ Дж } выражение р А Б "=" Дж п Дж | ψ Дж ψ Дж | не обязательно должен быть задан диагональной матрицей с собственными значениями п я по его диагонали. Тем не менее (как я считаю) всегда существует такой ортонормированный базис, который удовлетворяет вышеупомянутому условию. Например, мы можем взять для | ψ к столбец, в котором только k-й элемент не равен нулю и равен 1.
Теперь определим ортонормированные базисы { | а я } и { | б Дж } точно таким же образом. Тогда (мне кажется) у нас есть следующая связь:

| ψ Дж "=" с я , к Дж | а я | б к
где я не подразумеваю суммирование я и к . с я , к Дж "=" 1 для каждой комбинации я , к и Дж . Для этого весьма специфичного выбора базисов должно выполняться (1).

Тем не менее я подразумеваю, что мои рассуждения о восстановлении р А Б от р А и р Б должно держаться, потому что:

  • набор собственных значений не зависит от выбора базиса
  • матрица плотности полностью (?) определяется набором собственных значений
физика.stackexchange.com/ questions/1491/… этот вопрос, кажется, связан с моим, но мне кажется, что автор обсуждает некоторые другие аспекты.
Ваше выражение в ( 1 ) крайне сомнительно и, по крайней мере, неочевидно. Что заставляет вас думать, что р А Б можно разложить таким образом?
Я подозреваю, что правая часть (1) должна быть я , я , к , к С я , я , к , к | а я а я | | б к б к | , как следует из записи | ψ Дж "=" я , к с я , к Дж | а я | б к и ψ Дж | "=" я , к с я , к Дж , * а я | б к | . И, конечно, такое разложение не единственно... (мы можем выбрать любые ортонормированные базисы ( а я ) я и ( б к ) к мы хотим)
@Luzanne Ну, трудно полностью закрыть дверь из-за существования какой-то особой основы, которая снимет некоторую сложность с вашего общего выражения. (Но в данном случае я не думаю, что это выполнимо, или, по крайней мере, не сводится к выражению Ярослава.)
@EmilioPisanty Если ( а я ) я и ( б к ) к являются ортонормированными базисами , как утверждается, то (1) в его нынешнем виде будет означать, что | а я | б к образуют основу собственных состояний для р . Это очевидно невозможно, например, для | Ψ Ψ | (используя обозначения из вашего ответа), bcs собственное пространство для собственного значения 1 это 1-мерное пространство Охватывать { | Ψ } который не может быть записан в виде промежутка ряда | а я | б к .
@Luzanne Я думал о выборе локальных собственных базисов для матриц с уменьшенной плотностью, что могло бы упростить ситуацию. (В контексте квантовой химии их часто называют «естественными орбиталями».) Но я согласен, здесь это не сработает.

Ответы (2)

Оставляя в стороне некоторые сомнительные стороны вашего вопроса (в частности, тот факт, что ваше разложение в ( 1 ) это почти наверняка невозможно), есть простой ответ на основной вопрос, который вы ставите:

Итак, мой вопрос: учитывая спектры (наборы собственных значений) р А и р Б , можно ли вообще восстановить спектр р А Б ?

Нет , это невозможно. Это легко увидеть, сравнив

  • р А Б ( 1 ) "=" | Ψ Ψ | где | Ψ максимально запутанное состояние на двух кубитах, и
  • р А Б ( 2 ) "=" 1 4 я 4 , максимально смешанное состояние двух кубитов.

Оба они имеют идентичные приведенные матрицы плотности, р А "=" р Б "=" 1 2 я 2 с собственными значениями ( 1 , 1 ) , но спектр р А Б ( 1 ) является ( 1 , 0 , 0 , 0 ) и спектр р А Б ( 2 ) является ( 1 4 , 1 4 , 1 4 , 1 4 ) .

Чтобы дополнить чистый контрпример, данный Эмилио Писанти , и попытаться ответить на

Если нет, то стоит ли за этим результатом какая-то физическая интуиция?

часть вопроса, стоит отметить, что то же самое уже верно в классической статистической физике.

Если у меня есть распределение вероятностей п ( а , б ) для состояния составной классической системы невозможно восстановить это совместное распределение вероятностей из маргинальных распределений п 1 ( а ) "=" п ( а , б ) г б и п 2 ( б ) "=" п ( а , б ) г а . Что такое совместное распределение п кодирует то, что не захватывается п 1 и п 2 в одиночку, является знанием того, как состояние системы 1 коррелирует с системой 2 .

В квантовом случае, если мы знаем только матрицы парциальной плотности для каждой системы, теряются как классические, так и квантовые корреляции между двумя системами.